Đến nội dung

Hình ảnh

Topic bất đẳng thức THCS (2)


  • Chủ đề bị khóa Chủ đề bị khóa
Chủ đề này có 1115 trả lời

#261
dark templar

dark templar

    Kael-Invoker

  • Hiệp sỹ
  • 3788 Bài viết

Bài 128. (khá đơn giản)
Cho các số thực dương $a, b, c$ thoả mãn $a + b + c = 1$ . Chứng minh rằng
$$\sqrt{\dfrac{ab}{c + ab}} + \sqrt{\dfrac{bc}{a + bc}} + \sqrt{\dfrac{ca}{b + ca}} \le \dfrac{3}{2}$$

Đưa BĐT về dạng thuần nhất sau:
$$\sum \sqrt{\frac{ab}{(c+a)(c+b)}} \le \frac{3}{2}$$
Với cách đặt $x=\sqrt{\frac{ab}{(c+a)(c+b)}};y=...$,chúng ta đưa về 1 bài toán quen thuộc:
Bài toán : Cho $x,y,z>0$ thỏa mãn :$x^2+y^2+z^2+2xyz=1$.Chứng minh rằng:
$$x+y+z \le \frac{3}{2}$$

Bài viết đã được chỉnh sửa nội dung bởi Cao Xuân Huy: 20-01-2012 - 22:14

"Do you still... believe in me ?" Sarah Kerrigan asked Jim Raynor - Starcraft II:Heart Of The Swarm.

#262
Mai Duc Khai

Mai Duc Khai

    Thiếu úy

  • Thành viên
  • 617 Bài viết
Bài 129: Cho $a,b,c \in \left[ {0;1} \right]$
CMR:\[{\left( {1 + a + b + c} \right)^2} \ge 4\left( {{a^2} + {b^2} + {c^2}} \right)\]
P/s: Mọi người làm ác quá---Zz

___
Bài 129 cũ có nhiều trên diễn đàn rồi.

Bài viết đã được chỉnh sửa nội dung bởi Cao Xuân Huy: 20-01-2012 - 22:14

Tra cứu công thức toán trên diễn đàn


Học gõ Latex $\to$ Cách vẽ hình trên VMF


Điều mà mọi thành viên VMF cần phải biết và tuân thủ

______________________________________________________________________________________________

‎- Luật đời dạy em cách Giả Tạo
- Đời xô ... Em ngã
- Đời nham ... Em hiểm

- Đời chuyển ... Em xoay

Đời cay ... Em đắng


#263
Tham Lang

Tham Lang

    Thượng úy

  • Thành viên
  • 1149 Bài viết

Đưa BĐT về dạng thuần nhất sau:
$$\sum \sqrt{\frac{ab}{(c+a)(c+b)}} \le \frac{3}{2}$$

bài này đến đây còn một cách nữa cũng khá đơn giản là dùng AM-GM.

Off vĩnh viễn ! Không ngày trở lại.......


#264
dark templar

dark templar

    Kael-Invoker

  • Hiệp sỹ
  • 3788 Bài viết

bài này đến đây còn một cách nữa cũng khá đơn giản là dùng AM-GM.

Phân tích đơn giản như thế này mà thôi :D
$$\sqrt{\frac{ab}{(c+a)(c+b)}} \le \frac{1}{2}\left(\frac{a}{c+a}+\frac{b}{c+b} \right)$$
Nhưng ý của mình là các bạn hãy thử nhìn vào cái gốc của bài toán sẽ thấy rất thú vị :D,Bài toán mà mình đã nêu là 1 bài như thế,xung quanh nó có nhiều cái để bàn lắm ;)
"Do you still... believe in me ?" Sarah Kerrigan asked Jim Raynor - Starcraft II:Heart Of The Swarm.

#265
Tham Lang

Tham Lang

    Thượng úy

  • Thành viên
  • 1149 Bài viết

Bài 129:Cho $a,b,c \in \left[ {0;1} \right]$
CMR:\[{\left( {1 + a + b + c} \right)^2} \ge 4\left( {{a^2} + {b^2} + {c^2}} \right)\]
P/s: Mọi người làm ác quá---Zz

___
Bài 129 cũ có nhiều trên diễn đàn rồi.

$$(1 + a + b + c)^2 \ge 4(a + b + c) $$ nên ta cần cm $a + b + c \ge a^2 + b^2 + c^2 (1) $ Thật vậy $a(1 - a) \ge 0, b(1 - b) \ge 0, c(1 - c) \ge 0 $ nên (1) đúng.
Mình quên mất . Dấu "=" xảy ra khi t $(a, b, c)$ là hoán vị của $(0, 0, 1)$

Bài viết đã được chỉnh sửa nội dung bởi huymit_95: 20-01-2012 - 22:07

Off vĩnh viễn ! Không ngày trở lại.......


#266
Secrets In Inequalities VP

Secrets In Inequalities VP

    Sĩ quan

  • Thành viên
  • 309 Bài viết
Bài 130:Với 0<a,b,c<1.CMR:$\sum \frac{1-a}{1+b+c}\geq 3(1-a)(1-b)(1-c)$

__
MOD: Bạn kiểm tra lại đề đi nhé. Bài này sẽ xoá vào ngày mai nếu như đề không sửa lại.

Sorry, mình nhầm ,mình sửa lại rùi nè!

Bài viết đã được chỉnh sửa nội dung bởi Secrets In Inequalities VP: 21-01-2012 - 07:34


#267
Secrets In Inequalities VP

Secrets In Inequalities VP

    Sĩ quan

  • Thành viên
  • 309 Bài viết
Bai nua nha!

Bài 131:Cho a,b,c > 0.CMR: $\frac{b+c}{a}+\frac{2a+c}{b}+\frac{4(a+b)}{a+c}\geq 9$

___
MOD: Vui lòng Đánh tiếng Việt có dấu!

Bài viết đã được chỉnh sửa nội dung bởi Cao Xuân Huy: 20-01-2012 - 22:42


#268
Tham Lang

Tham Lang

    Thượng úy

  • Thành viên
  • 1149 Bài viết
Mình làm luôn
$$\dfrac{b + c}{a} + \dfrac{2a + c}{b} + \dfrac{4(a + b)}{a + c} = \dfrac{b}{a} + \dfrac{c}{a} + \dfrac{a + c}{b} + \dfrac{a}{b} + \dfrac{4a}{a + c} + \dfrac{4b}{a + c} $$ $$= (\dfrac{b}{a} + \dfrac{a}{b}) + (\dfrac{a + c}{b} + \dfrac{4b}{a + c}) + (\dfrac{c + a}{a} + \dfrac{4a}{a + c} - 1) \ge 2 + 4 + (4 - 1) = 9$$ (sử dụng côsi)

Bài viết đã được chỉnh sửa nội dung bởi huymit_95: 20-01-2012 - 22:39

Off vĩnh viễn ! Không ngày trở lại.......


#269
Tham Lang

Tham Lang

    Thượng úy

  • Thành viên
  • 1149 Bài viết

Bài 130:Với a,b,c>0.CMR:$\sum \frac{1-a}{1+b+c}\geq (1-a)(1-b)(1-c)$

Mình thấy đề bài này không ổn chút nào ! Mọi người hãy cùng mình thử với $a = b = 2, c = \dfrac{1}{2}$ nhé (có thể mình thử sai) Khi đó, VT < 0, VP > 0. Mình nghĩ bài này cần thêm điều kiện

Off vĩnh viễn ! Không ngày trở lại.......


#270
Tham Lang

Tham Lang

    Thượng úy

  • Thành viên
  • 1149 Bài viết
Mình xin góp thêm một số bài.
Bài 131. Cho các số thực dương $a, b, c$. Chứng minh bất đẳng thức
$$(abc + 1)(\dfrac{1}{a(b + 1)} + \dfrac{1}{b(1 + c)} + \dfrac{1}{c(1 + a)}) \ge \dfrac{(a^5 + b^5 + c^5)(a^3 + b^3 + c^3)}{a^8 + b^8 + c^8}$$
Bài 132. Cho $a, b$ là các số thực dương thoả mãn $a + b \le 1$ . Chứng minh rằng
$$\dfrac{1}{a^3 + b^3} + \dfrac{1}{a^2b} + \dfrac{1}{ab^2} \ge 20$$
Bài 133. Cho $\dfrac{1}{2} \le a, b, c \le 1$ . Chứng minh rằng
$$2 \le \dfrac{a + b}{1 + c} + \dfrac{b + c}{1 + a} + \dfrac{c + a}{1 + b} \le 3$$
Bài 134.Cho $a, b, c$ là độ dài 3 cạnh của một tam giác. Chứng minh rằng
$$(a + b + c)(\dfrac{1}{a} + \dfrac{1}{b} + \dfrac{1}{c}) + \dfrac{3(a - b)(b - c)(c - a)}{abc} \ge 9$$
Bài 135. Cho các số thực $a_1, a_2, ..., a_n $ . Chứng minh rằng
$$\sqrt[3]{a^3_1 + a^2_2 + ... + a^3_n} \le \sqrt{a^2_1 + a^2_2 + ... + a^2_n}$$

Off vĩnh viễn ! Không ngày trở lại.......


#271
Ispectorgadget

Ispectorgadget

    Nothing

  • Quản lý Toán Phổ thông
  • 2946 Bài viết
Bài 132. Cho $a, b$ là các số thực dương thoả mãn $a + b \le 1$ . Chứng minh rằng
$$\dfrac{1}{a^3 + b^3} + \dfrac{1}{a^2b} + \dfrac{1}{ab^2} \ge 20$$
$VT=\frac{1}{a^3+b^3}+\frac{1}{2a^2b}+\frac{1}{2ab^2}+\frac{1}{2a^2b}+\frac{1}{2ab^2}\geq \frac{25}{(a+b)^3+ab(a+b)}\geq \frac{25}{(a+b)^3+\frac{(a+b)^3}{4}}\geq 20$

Dấu "=" xảy ra khi a=b=$\frac{1}{2}$
Mai xử tiếp giờ đi Zz.

►|| The aim of life is self-development. To realize one's nature perfectly - that is what each of us is here for. ™ ♫


#272
dark templar

dark templar

    Kael-Invoker

  • Hiệp sỹ
  • 3788 Bài viết

Mình xin góp thêm một số bài.
Bài 131. Cho các số thực dương $a, b, c$. Chứng minh bất đẳng thức
$$(abc + 1)(\dfrac{1}{a(b + 1)} + \dfrac{1}{b(1 + c)} + \dfrac{1}{c(1 + a)}) \ge \dfrac{(a^5 + b^5 + c^5)(a^3 + b^3 + c^3)}{a^8 + b^8 + c^8}$$

Gợi ý: Chứng minh BĐT kép sau:
$$(abc+1)\left[\frac{1}{a(b+1)}+\frac{1}{b(c+1)}+\frac{1}{c(a+1)} \right] \ge 3 \ge \frac{(a^5+b^5+c^5)(a^3+b^3+c^3)}{a^8+b^8+c^8}$$
Có thể thấy VP,ta có thể thay bằng đại lượng $\frac{(a^{m}+b^{m}+c^{m})(a^{n}+b^{n}+c^{n})}{a^{k}+b^{k}+c^{k}}$ trong đó $m,n,k \in \mathbb{N^*};m+n=k$
"Do you still... believe in me ?" Sarah Kerrigan asked Jim Raynor - Starcraft II:Heart Of The Swarm.

#273
dark templar

dark templar

    Kael-Invoker

  • Hiệp sỹ
  • 3788 Bài viết

Bài toán : Cho $x,y,z>0$ thỏa mãn :$x^2+y^2+z^2+2xyz=1$.Chứng minh rằng:
$$x+y+z \le \frac{3}{2}$$

Nhưng ý của mình là các bạn hãy thử nhìn vào cái gốc của bài toán sẽ thấy rất thú vị :D,Bài toán mà mình đã nêu là 1 bài như thế,xung quanh nó có nhiều cái để bàn lắm ;)

Giờ mới kiếm được link cho các em tham khảo những cái nhìn mở rộng về bài toán trên:
http://diendantoanho...l=&fromsearch=1
Và những cách chứng minh khác nhau cho bài toán:
http://diendantoanho...showtopic=65361
Và cũng là ý tưởng để anh sáng tạo ra bài toán dưới đây:
http://www.artofprob...uality#p2316849

Bài viết đã được chỉnh sửa nội dung bởi dark templar: 21-01-2012 - 07:56

"Do you still... believe in me ?" Sarah Kerrigan asked Jim Raynor - Starcraft II:Heart Of The Swarm.

#274
Mai Duc Khai

Mai Duc Khai

    Thiếu úy

  • Thành viên
  • 617 Bài viết
Bài 136:(Dễ) Cho $a,b,c>0$ thỏa mãn: ${a^2} + {b^2} + {c^2} = 4\sqrt {abc}$
Chứng minh BĐT sau bằng nhiều cách: \[a + b + c > 2\sqrt {abc} \]
Ta còn có 2 bài toán tổng quát hơn:
Bài toán 1: Cho n số dương ${a_1},{a_2},...,{a_n}$ thỏa mãn:
\[a_1^m + a_2^m + ... + a_n^m = p\sqrt {{a_1}{a_2}...{a_n}} \left( {1 \le m,p < 2n} \right)\]
CMR:\[a_1^{n - m} + a_2^{n - m} + ... + a_n^{n - m} > (2n - p)\sqrt {{a_1}{a_2}...{a_n}} \]
Bài toán 2: Cho n số dương ${a_1},{a_2},...,{a_n}$ thỏa mãn:
\[a_1^{n - 1} + a_2^{n - 1} + ... + a_n^{n - 1} = p\sqrt {{a_1}{a_2}...{a_n}} \]
CMR:\[{a_1} + {a_2} + ... + {a_n} \ge \frac{{{n^2}}}{p}\sqrt {{a_1}{a_2}...{a_n}} \]
ZzzzzzzzzzzzzZ

Bài viết đã được chỉnh sửa nội dung bởi Cao Xuân Huy: 21-01-2012 - 11:50

Tra cứu công thức toán trên diễn đàn


Học gõ Latex $\to$ Cách vẽ hình trên VMF


Điều mà mọi thành viên VMF cần phải biết và tuân thủ

______________________________________________________________________________________________

‎- Luật đời dạy em cách Giả Tạo
- Đời xô ... Em ngã
- Đời nham ... Em hiểm

- Đời chuyển ... Em xoay

Đời cay ... Em đắng


#275
Ispectorgadget

Ispectorgadget

    Nothing

  • Quản lý Toán Phổ thông
  • 2946 Bài viết
Bài 135. Cho các số thực $a_1, a_2, ..., a_n $ . Chứng minh rằng
$$\sqrt[3]{a^3_1 + a^2_2 + ... + a^3_n} \le \sqrt{a^2_1 + a^2_2 + ... + a^2_n}$$

Để đơn giản ta viết $\sum\limits_{i = 1}^n {a_i^2 } = a_1^2 + a_2^2 + ... + a_n^2$ (chắc nhiều người biết rồi mà thôi cứ viết ra cho rõ)
Đặt VP =x $(x\geq 0)$
$ \Rightarrow x^2 = \sqrt {\sum\limits_{i = 1}^n {a_1^2 } }$

Nếu x=0 thì dấu "=" xảy ra.
Nếu x>0 thì $(\frac{a_1}{x})^2+(\frac{a_2}{x})^2+...+(\frac{a_n}{x})^2=1$
$(\frac{a_1}{x})^2\leq 1;(\frac{a_2}{x})^2\leq 1;...(\frac{a_n}{x})^2\leq 1;$
$\Rightarrow |\frac{a_1}{x}|\leq 1;|\frac{a_2}{x}|\leq 1;...|\frac{a_n}{x}|\leq 1$
$\Rightarrow (\frac{a_1}{x})^3\leq (\frac{a_1}{x})^2;...;(\frac{a_n}{x})^3\leq (\frac{a_n}{x})^2;$
$\Rightarrow (\frac{a_1}{x})^3+...+(\frac{a_n}{x})^3\leq (\frac{a_1}{x})^2+...+(\frac{a_n}{x})^2=1$
Từ đây ta có:$a_2^3+a_1^3+...+a_n^3\leq x^3\Rightarrow \sqrt[3]{a_2^3+a_1^3+...+a_n^3}\leq x=VP$ (đpcm) Zz

Bài viết đã được chỉnh sửa nội dung bởi Ispectorgadget: 21-01-2012 - 10:26

►|| The aim of life is self-development. To realize one's nature perfectly - that is what each of us is here for. ™ ♫


#276
nguyenta98

nguyenta98

    Thượng úy

  • Hiệp sỹ
  • 1259 Bài viết

Bài 134.Cho $a, b, c$ là độ dài 3 cạnh của một tam giác. Chứng minh rằng
$$(a + b + c)(\dfrac{1}{a} + \dfrac{1}{b} + \dfrac{1}{c}) + \dfrac{3(a - b)(b - c)(c - a)}{abc} \ge 9$$

Bài 134:
$(a+b+c)(\dfrac{1}{a}+\dfrac{1}{b}+\dfrac{1}{c})+\dfrac{3(a-b)(b-c)(c-a)}{abc}=\dfrac{(a+b+c)(ab+bc+ca)+3(a-b)(b-c)(c-a)}{abc}$ <1>
Lại thấy $(a+b+c)(ab+bc+ca)+3(a-b)(b-c)(c-a)=3abc+2(ab^2+bc^2+ca^2)\geq 3abc+2\sqrt[3]{ab^2bc^2ca^2}=9abc$ <2>
Từ <1>;<2> suy ra $đpcm$ Dấu $"=" \leftrightarrow a=b=c$ hay tam giác ABC là đều

Bài viết đã được chỉnh sửa nội dung bởi nguyenta98: 21-01-2012 - 10:44


#277
Ispectorgadget

Ispectorgadget

    Nothing

  • Quản lý Toán Phổ thông
  • 2946 Bài viết

Gợi ý: Chứng minh BĐT kép sau:
$$(abc+1)\left[\frac{1}{a(b+1)}+\frac{1}{b(c+1)}+\frac{1}{c(a+1)} \right] \ge 3 \ge \frac{(a^5+b^5+c^5)(a^3+b^3+c^3)}{a^8+b^8+c^8}$$
Có thể thấy VP,ta có thể thay bằng đại lượng $\frac{(a^{m}+b^{m}+c^{m})(a^{n}+b^{n}+c^{n})}{a^{k}+b^{k}+c^{k}}$ trong đó $m,n,k \in \mathbb{N^*};m+n=k$

Em giải nốt vế sau luôn :P
Không mất tính tổng quát giả sử

$a\geq b\geq c$
$\Rightarrow a^5\geq b^5\geq c^5;a^3\geq b^3\geq c^3$
Áp dụng BĐT Chebyshev cho bộ dãy đơn điệu tăng
Ta có: $a^5.a^3+b^5.b^3+c^5.c^3\geq \frac{(a^5+b^5+c^5)}{3}(a^3+b^3+c^3)$
Từ đây suy ra đpcm
Còn vế đầu mình có chứng minh trong box này rồi các bạn có thể tham khảo thêm ^_^ Zz

Bài viết đã được chỉnh sửa nội dung bởi Ispectorgadget: 21-01-2012 - 11:02

►|| The aim of life is self-development. To realize one's nature perfectly - that is what each of us is here for. ™ ♫


#278
Tham Lang

Tham Lang

    Thượng úy

  • Thành viên
  • 1149 Bài viết

Bài 130:Với 0<a,b,c<1.CMR:$\sum \frac{1-a}{1+b+c}\geq 3(1-a)(1-b)(1-c)$

__
MOD: Bạn kiểm tra lại đề đi nhé. Bài này sẽ xoá vào ngày mai nếu như đề không sửa lại.

Sorry, mình nhầm ,mình sửa lại rùi nè!

Với nài này, mình mạo muội sửa chút ở đề :) : $0\le a, b, c\le1$
TH1. Nếu ít nhất 1 trong 3 số $a, b, c$ bằng 0 thì $VT \ge 0, VP = 0 $ Nên BĐT đúng
TH2. Nếu cả 3 số $a, b, c$ đều khác 1.
$$BĐT \Leftrightarrow \dfrac{1}{(1 + b + c)(1 - b)(1 - c)} + \dfrac{1}{(1 + a + c)(1 - a)(1 - c)} + \dfrac{1}{(1 + a + b)(1 - a)(1 - b)} \ge 3$$ Ta lại có : $$(1 + b + c)(1 - b)(1 - c) \le \dfrac{(1 + b + c + 1 - b + 1 - c)^3}{27} = 1 \Leftrightarrow \dfrac{1}{(1 + b + c)(1 - b)(1 - c)} \ge 1$$Tương tự với các bộ số khác, cộng vế theo vế suy ra đpcm. Dấu "=" xảy ra khi $a = b = c = 0$ hoặc$a = b = c = 1$. Không biết mình sửa đề như vậy có vi phạm không nữa nhưng khi cho $a = b = c$ thì chúng không thuộc $(0, 1)$. Mong được sự góp ý của mọi người. Còn nếu giữ nguyên đề ra thì chẳng xảy ra dấu "="

Bài viết đã được chỉnh sửa nội dung bởi huymit_95: 21-01-2012 - 13:45

Off vĩnh viễn ! Không ngày trở lại.......


#279
Cao Xuân Huy

Cao Xuân Huy

    Thiếu úy

  • Hiệp sỹ
  • 592 Bài viết

Bài 136:(Dễ) Cho $a,b,c>0$ thỏa mãn: ${a^2} + {b^2} + {c^2} = 4\sqrt {abc}$
Chứng minh BĐT sau bằng nhiều cách: \[a + b + c > 2\sqrt {abc} \]

Máy không có mathtype khó gõ $\LaTeX$ quá

Ta có bđt tương đương:

$$(a^2+b^2+c^2)(a+b+c) \ge 8abc$$

Áp dụng bđt AM-GM ta có:

$$(a^2+b^2+c^2)(a+b+c) \ge 3.\sqrt[3]{a^2b^2c^2}.3.\sqrt[3]{abc}=9abc \ge 8abc$$

Vậy ta có ĐPCM.


Cũng cách giải tương tự ta chứng minh được 2 bài tổng quát

Dấu "=" xảy ra khi $a=b=c=0$

Mà $a,b,c >0$ nên dấu "=" không xảy ra.

ZzzzzzZ

Bài viết đã được chỉnh sửa nội dung bởi Cao Xuân Huy: 21-01-2012 - 11:52

Cao Xuân Huy tự hào là thành viên VMF

Hình đã gửi


#280
Ispectorgadget

Ispectorgadget

    Nothing

  • Quản lý Toán Phổ thông
  • 2946 Bài viết
Bài 137: (không biết dễ hay khó :lol: Zz )
Quy nạp nhé ;)
Cho n số thực dương $x_1;x_2;...x_n$ có tích bằng 1. Chứng minh rằng
$x_1+x_2+...+x_n\geq n$

Bài viết đã được chỉnh sửa nội dung bởi Ispectorgadget: 21-01-2012 - 12:25

►|| The aim of life is self-development. To realize one's nature perfectly - that is what each of us is here for. ™ ♫





1 người đang xem chủ đề

0 thành viên, 1 khách, 0 thành viên ẩn danh